Platonism and the non-physical

On occasion, I’ve been accused of being closed-minded.  (Shocking, I know.)  Frequently the reason is not seriously considering non-physical propositions, a perception of rigid physicalism.  However, as I’ve noted before, I’m actually not entirely comfortable with the “physicalist” label (or “materialist”, or other synonyms or near synonyms).  While it’s fairly accurate as to my working assumptions, it actually doesn’t represent a fundamental commitment.

My actual commitment is empiricism.  By “empiricism” here, I don’t necessarily mean physical measurement, but conscious experience, specifically reproducible or verifiable experience, and inferred theories that can predict future experiences, with an accuracy better than alternate theories, or at least better than random chance.  I do generally assume physicalism is true, mainly because many physical propositions seem able to meet this standard, where non-physical ones seem to struggle with it.

But that raises a question.  Are there any non-physical propositions that do meet the standard?  It depends on what we’re willing to consider non-physical.   In the Chalmers post a few weeks ago, I noted that we could interpret his views in a platonic or abstract fashion, in which case the differences between him and a functionalist might collapse into differences in terminology.  Although as I also noted, neither Chalmers nor Dennett would agree.

And this bridge between the views depends on your attitude toward platonism.  Note that “platonism” with a small ‘p’ doesn’t really refer to the philosophy of Plato, but to a modern outlook that regards abstract concepts as real.  This is sometimes described as real in a separate platonic realm, which many misinterpret as meaning a physical existence in a parallel universe or something.

But in modern platonism, abstract objects are held to have no spatio-temporal properties, and to be causally inert.  If they have an existence, it is one completely separate from time and space.  It’s not even right to say they’re “outside” of time and space, because that implies a physical location, something they don’t have.

What are examples of these abstract objects?  Numbers, mathematical relations, properties such as redness, structures, patterns, etc.  Under platonism, these things are held to have a non-physical existence.  For the Chalmers outlook, the property one is important since he often refers to his view as property dualism.

But is platonism true?  One of the the strongest arguments for it appears to be the way we talk about abstract objects.  We refer to concepts like “7” as though they have an existence separate and apart from a pattern of seven objects.  We refer to structures and properties in much the same way.  The fact that we can discuss “redness” coherently seems to imply we accept that property as having an independent existence.

But this assumes that analyzing language is in any way meaningful for what’s real.  At best, it might just show our intuitions, intuitions we might not even believe.  For instance, we refer to things like the sun “rising” and “setting” all the time without seriously thinking that the sun is moving around us (at least since Copernicus and Galileo).  It might be that all this usage should be viewed as metaphorical, and abstract objects as “useful fictions”.

But the dividing line between a useful fiction and a real concept seems like a blurry one.  The more useful a concept is, particularly one useful in an epistemic fashion, the harder it seems to dismiss as a fiction.  We reach a point where we have to invest a lot of energy in explaining why it’s not real.

That said, a strong case against platonism is also an epistemic one.  If minds exist in this universe, and abstract objects exist without any spatio-temporal aspects, and are causally inert, how can we know about them?  We could say the mind is capable of accessing abstract objects, but this implies something super-physical about it.  The relevant physics appear to be causally closed, and this proposition wouldn’t meet the empiricism criteria above.

The more usual defense is that we infer the existence of abstract objects by what we observe in the physical world, by the patterns and relations we see there.  But if that’s how we come to know about abstract objects, why do we actually need the separate abstract objects themselves?  Why can’t we just get by with the models in our mind and the physical patterns they’re based on?

This last point has long been what makes me leery of platonism.  A ruthless application of Occam’s razor seems to make it disappear in a flash of parsimony.  It doesn’t seem necessary.  And given how far some people have tried to run with it, this seems important.

All that said, this is a case where I’m not confident in my conclusion, at least not yet.  I still wonder if its pragmatic value might not imply ontology.  Everything in physics above the level of fundamental forces and quantum fields seems to exist as structure and function, a pattern of lower level constituents.

In many cases, these structures and functions, such as wings or the shape of fish, seem convergent.  These convergences could be seen as implying that the converged structure has an independent reality.  Of course, these are optimal energy structures that emerge from the laws of physics, but then do the laws themselves have an independent reality aside from the physical patterns and regularities?  Are they themselves abstract entities?

And the fact that large portions of the mathematics profession are mathematical platonists gives me pause.  Mathematicians seem convinced that they’re discovering something, not developing tools in some nominalist sense, although the dividing line between invention and discovery itself seems pretty blurry.

If platonism is true, then we have a non-physical reality, and properties such as consciousness (the property of being conscious) could be said to exist non-physically in a platonic sense.  To be sure, this is a far more limited sense of non-physical than many advocates of dualism envision.

Interestingly, Chalmers himself does not appear to be a platonist, but appears to consider the question of the existence of abstract objects to have no fact of the matter answer, espousing a view called ontological anti-realism.  Given my own instrumentalist leanings, I may have to investigate this view.  But it also implies my attempt at steel-manning his argument is probably fruitless.

What do you think?  Do you see other arguments for platonism?  Or against?  Or is the whole thing just hopeless navel-gazing?

146 thoughts on “Platonism and the non-physical

  1. SelfAwarePatterns,

    What is your definition or meaning of ‘existence’? How does existence differ from non-existence? Can there be existence which is not only unknown to any human but is unknowable to any human?

    Like

    1. My initial thought is that existence involves being real in some objective sense, that is, having being independent of mind. Admittedly, it’s very difficult to define a word like “existence” without using synonyms.

      Can there be existence that is unknown to humans? Sure. There are plenty of things that exist that we only became aware of in the last few centuries.

      Are there things that are unknowable to any human? There probably are, although given the history of science, it doesn’t seem productive to make assumptions about particular topics. Some things that were once hopelessly metaphysical, like atoms, eventually became scientific phenomena. Someone once said we’d never know what the stars were made of, not that long before the discovery of spectral absorption lines. So statements of unknowability should definitely be considered provisional.

      Liked by 1 person

  2. [donning waders for deep water]
    These issues are why I try to be more circumspect with the use of the term “exist”.

    My working axioms are:
    1. physical stuff exists
    2. patterns are real
    3. things change

    So “exists” only applies to physical things, but “reality” applies to abstractions. When we ask if a physical thing is real, we’re asking if there is a thing that exists and exhibits a (real) pattern, because only things that exist exhibit patterns.

    *
    [prepping scuba gear]

    Like

    1. So, abstract objects are real, but don’t exist? That seems like a strange statement. I’m not going to quibble with your definition of “exist” since my own isn’t super well thought out. But I’m not sure the real / exist distinction meets with the common use of the words.

      Like

      1. The statement seems strange because the concept of object, without more, implies an existing thing, which makes “abstract object” have a self-contradictory flavor.

        I’m not saying all my uses of terms will be consistent with common usage, because nobody’s use of terms can be consistent with common usage and also be unambiguous. Common usage is ambiguous. So I will choose to be self-consistent and unambiguous.

        Thus, an abstract object is a real pattern that could potentially be exhibited by an existing object.

        *

        Liked by 1 person

  3. “My initial thought is that existence involves being real in some objective sense, that is, having being independent of mind.’

    o.k, So I gather that you are saying that if X has a being without mind then it has a being.

    Does mind have a being?

    Like

    1. On what it is for something to exist or be real: I am of the view that existence is relative, i.e. it is mutual between two things. Something exists to something else to the extent that it interacts with it (directly or indirectly). So we should never really talk of ‘exists’ or ‘is real’ without mentioning what the two participants are.

      This derives from thinking about there hypothetically existing other dimensions, in no way connected with ours, in which other things could exist to each other, but never exist for us.

      For something to just exist or be real without reference to what it can interact with (which is the sense in which we usually ask if things are real), we have to put in place a god, or god-like physicist to interact with, or speculate that potentially it could interact with any of us in a similar way, which I guess is what Science tries to do. That’s a shorthand though, that masks who the 2 participants are in the ‘exists to’ relationship.

      In terms of the existence or reality of things I am subjectively conscious of, there is then no problem because they exist and are real for me, but I need to take care in defining what I really am – a mental thing linked out into physical space.

      Liked by 1 person

      1. Peter, I agree with the essence of the metaphysics, but I disagree with the statement “we should never really talk of ‘exists’ or ‘is real’ without mentioning what the two participants are.” For most things which exist, the number of other objects which it can “exist to” is vast. I choose to say that it exists if there is at least one thing it can “exist to”. Consider neutrinos. We weren’t sure they existed until we could find another thing that we were pretty sure they “existed to” in a reliable way. (Vast pool of clear water mile or so underground). Nevertheless, neutrinos can “exist to” your body. It’s just doesn’t happen very often.

        *

        Liked by 1 person

        1. James, ‘at least one thing it exists to’ doesn’t seem to work because of the example that if things existed to each other in dimensions inaccessible to ours they still wouldn’t exist to me. There may be interesting things to explore about a sort of algebra of existence: if A exists to B does B necessarily exist to A. And if X exists to Y and Y exists to Z, does it necessarily follow that X exists to Z?

          Like

          1. Just a reminder, I said at least one thing it can exist to.

            I don’t see the point of thinking about other dimensions. If it doesn’t interact with something in my dimension, I really don’t care. It will never affect me.

            As for your algebra of existence, I have a gut instinct that quantum mechanics is where this algebra is getting worked out.
            “Existing to” = measurement = entanglement.
            Just guessing, though.

            *

            Like

  4. Great post!

    This is why I like to think in terms of testability. Rather than speaking of existence, I like to speak of what we can verify. If we can’t test X, then I simply avoid committing to any position regarding X.

    I think in some sense, mathematicians discover something, in the same sense that one can discover an error in a piece of reasoning, or even discover a structure to an argument. In this case, the realm traversed to find the discovery is not physical, but axiomatic (or linguistic if you prefer).

    But I agree; when pressed, many mathematicians do speak about their fields as if they are platonists. One way to bring it out is to discuss the “existence” of infinity and to bring up the intuitionists and constructivists. Fun times 🙂

    Liked by 1 person

    1. Thanks BIAR!

      I can see where you’re coming from with the testability criteria, although I think we have to be careful about not letting it slide into verificationism, and the whole logical positivism trap.

      The question is, does the crucial nature of abstract concepts for theories that can predict future experiences, amount to a test of the existence of that concept? The concept certainly exists as a physical model in our minds, and there are physical patterns that fit it. Is that enough? Or does the obvious convenience of thinking of the concept in and of itself as existing amount to any kind of test?

      Liked by 1 person

      1. I think verificationism and logical positivism gets a bad rap, but I may think this simply because I adhere to a pragmatic version of it, so I never found the attacks on it particularly problematic.

        I wouldn’t think of it as a test for the concept. I’d think of the concept as a linguistic construct, that’s a short-hand for a variety of things, which are directly or indirectly testable. At some point, we can just realize that having these linguistic constructs, or useful fictions a valuable thing and keep using it as long as no test challenges the things that the concept applies.

        For instance, take something like Quantum Mechanics. We talk about a particle also behaving like a wave, so we’re using particles and waves as concepts. Now, the truth of what’s going on could be inconceivable, so these are just useful fictions, but insofar as they can be a short-hand for the observation of properties associated with particles along with properties associated with waves, then this concept is useful, even if it is not directly being proven. Likewise with mathematical structures. To talk about things like the wave function is to simply use a shorthand to talk about the mathematical pattern underlying an observed behavior. As long as we don’t cling to the concept and understand it as a tool, then it’s fine.

        But when we get into concepts that aren’t testable — like philosophies around QM (such as Many Worlds) then I think we should take a page out of the verificationist playbook. We can chat about it for fun, but there’s nothing that can seriously be done with such claims because they are not testable and as far as I can tell, they have no testable implications that would provide some confidence in those worldviews over others.

        Like

        1. I think verificationism is totally fine, even crucial, for empirical evidence. But for theories, it seems too stringent. Effective theories need intellectual space to have unverifiable components, which is why falsifiability coupled with parsimony, seems like a better standard. It lets standard scientific theories in. With strict verificationism, most of modern science doesn’t seem to make the cut. Or perhaps another way of putting it is, falsifiability for theories strikes me as pragmatic verificationism.

          Your talk about particles and waves being useful fictions seems to put you in the instrumentalist camp. If so, I’m there too. So I agree, they may well be metaphors we’re using to make sense out of something that our minds, minds evolved to deal with macroscopic dynamics, are simply incapable of grasping.

          On interpretations and testability, that’s the thing. None of them are yet testable. But that could change. I think of John Bell and his theorem, which was able to resolve what seemed like a hopelessly metaphysical issue. Today’s metaphysics could be tomorrow’s science. There’s no way to know ahead of time. All we can do is keep slamming against the epistemic wall and hope it cracks.

          Like

  5. My take is that Platonism is about mental discriminations that we make, which give rise to words, objects, colours, sets, numbers, maths and logic. These are physically underpinned by neural pattern-matching on attended data. In conscious reasoning we can access information about what we are attending to and which patterns we are matching (i.e. the categories themselves). In logical thought we can manipulate these in the abstract.

    I’ve been thinking about the extent to which physics, at its heart, makes no discriminations, and has no categories. Even the slightest difference in the smallest part of a solution to the equations of physics is an utterly different solution. Thus there is no common thing that is an electron, nor even the same electron persisting over time….these are just determinations that we make when we offer up our mental categories to the world and find they match sufficiently for our purposes. There are no categories in the physical world, even though we use categories to interpret the physical world so that we can act in it, and our brain provides physical mechanisms we use to hold and manipulate those categories.

    Liked by 1 person

    1. That seems close to nomimalism, which is my own tentative conclusion.

      I agree there are no categories in physics. But there are patterns and regularities, patterns and regularities that change over time. If we had every living creature that ever lived in front of us, there would be no species, just gradually changing forms. But as we look at the landscape today, with the vast majority of creatures that ever lived not present, there are wide gaps in the patterns, and what remains are what we call “species”.

      But ultimately I think you’re right. It all comes down to affordances, aspects of our environment that are relevant to our goals. To a virtual construct, a virtual castle is just as real as a physical castle is to us. The advantage we have over the virtual is that our environment causes theirs, but we shouldn’t get cocky. Maybe there’s an outer environment that causes ours.

      Liked by 1 person

  6. Great post (as usual)! I think Plato argued rather persuasively for the category of ideals (straight lines, triangles, circles, etc.) that are not dependent on minds for their existence. I’m ok with calling them patterns instead of ideals, but they seem somewhat different from properties. A triangle can have properties (3-sided) and can be a property itself (something may be triangular), but Plato’s ideal triangles don’t exist in reality. Reality may approximate perfection (and perfection may approximate reality). I suppose ideals are like statistical averages or means or various sigma points that may or may not actually represent an entity. I think the word “existence” is semantically overloaded. It may very well be that there is no physical reality above the level of quantum fluctuations but it is a fact that I, as a biological entity, can get up from my chair and move my fluctuating quanta from one place to another. As you have often stated, we are all patterns.

    Liked by 1 person

    1. Thanks Mike!

      “I suppose ideals are like statistical averages or means or various sigma points that may or may not actually represent an entity.”

      This fits with my own intuition. These concepts may exist as statistical simplifications, frameworks that our brains build in order to facilitate predictions. But they’re usefulness, and the fact that we all come up with the same ones, makes me wonder. But the actual reality may only be these brain models and the environmental patterns they’re based on.

      Liked by 1 person

  7. What if time and space are also abstract?

    The distinction between abstract and non-abstract would largely vanish. All properties would be abstract in a sense.

    Sure. Time and space seem more real or substantial than say the number “7”. But are they?

    Like

    1. In the way you’re using it, how would you define “abstract”?

      This line of thinking leads to Max Tegmark’s Mathematical Universe Hypothesis. I did a series of posts several years ago on his book on the subject. His basic premise is that it’s all structure, patterns, relations, all the way down, meaning that all of reality is math, not the notations and such, but the underlying mathematical reality.

      It’s that last point that makes me wonder if the notations aren’t referring to a mathematical reality, but reality, and derived fictions.

      Like

      1. Certainly time doesn’t exist except as an abstraction from records we have in the present and an imaginary potential future. The past is gone. The future hasn’t happen. All we have is the present.

        Motion, since is dependent on time, would be an abstraction. We have a record of an object at one location. It is now at another location. We abstract motion from the current observation and the record of a previous observation.

        In a sense, it is only from motion we derive a sense of space. In another sense Euclidean space is an arbitrary grid we impose on our perceptions of the world.

        Liked by 1 person

        1. That seems like a Newtonian conception of time and space. In Newton’s model, space is just a stage for causal events, and there is only the eternal now throughout the universe. But in Einstein’s model, which is more accurately predictive, spacetime is compressed and warped by matter and energy, and affects the path of matter in its traversal through it. Under special and general relativity, spacetime isn’t just the stage for causal events, it’s an active participant.

          Which raises an interesting question. Time could once have been considered an abstraction, but now is part of the causal structure. What else that is now an abstraction might, in the future, we become aware is part of the causal framework?

          Liked by 2 people

          1. Regardless of which mechanisms we might erroneously conclude to be a part of the causal framework, the question still remains: Are the perceptions which “we” impose upon the world and consider to be causal structures the true nature of reality? According to Parmenides’ assessment, the definitive answer is no.

            Like

          2. I don’t know that we have any choice but to “impose” things upon the world. It’s the only was we, as a primate from the African environments, can make sense of it in an adaptive manner. In the end, all we can do is develop theories, and see which are more or less predictive.

            Like

          3. In almost any sense time has to be an abstraction. The past no longer exists. All matter/location that did exist in the past now only exists in present. Whatever state it had in the past is now only a record in the present.

            I don’t know what you mean space time is a stage for causal events. Space time is an abstraction itself, a description that can predict various things better than Newtonian model but still an abstraction.

            Like

          4. James,
            Are you familiar with general relativity? In short, it posits that gravitation is the warping of spacetime. and that warping actually propagates at the speed of light. Spacetime is part of the physical causal framework. It affects and is affected by mass and energy.

            There are numerous consequences, but one of them is the relativity of simultaneity. https://en.wikipedia.org/wiki/Relativity_of_simultaneity
            Which is to say, there is no universal now. Einstein banished it, so you can’t really consign the past or future to nonexistence, or you need to regard that existence itself as relative.

            Of course, it’s possible that the actual reality isn’t spacetime warping, that it’s just a very successful metaphor, but if so, then any alternate model has to explain the same observations that general relativity does. General relativity is one of the most tested scientific theories of all time, so if it’s wrong, outside of very exotic conditions (black holes, big bang, etc), it’s going to be so in an extremely nuanced fashion.

            Liked by 1 person

          5. Einstein was describing the effects of gravity with an abstract spacetime model.

            That doesn’t somehow make spacetime more real or substantive. It is still abstract, described by mathematics, for God’s sake. Unless you think spacetime is actually some sort of material itself, kind of like an aether.

            “Which is to say, there is no universal now. Einstein banished it, so you can’t really consign the past or future to nonexistence, or you need to regard that existence itself as relative.”

            Exactly. Existence is relative to the observer like the now.

            The past and future cannot be physical; otherwise, we would have all of their physical states coexisting with the present physical states. They are abstractions. I don’t think I said they don’t exist. But, if I did, I mean to say they exist but they exist as abstractions. They are non-physical.

            Which raises the point: What exactly is the physical that you are contrasting with the platonic or non-physical reality?

            If you start to decompose what we normally think of as physical reality eventually you reach protons, electrons, etc or going lower still quarks. In essence the physical at its bottom isn’t really so much physical as it is just a bunch of measurable observations of something that sometimes we conceptualize as a particle and sometimes conceptualize as a wave. Relative to the observer, of course.

            Like

          6. On the physical, I discussed it a bit in my response to Wyrd below, but I also discussed it in a post a while back.
            https://selfawarepatterns.com/2016/06/06/what-is-physicalism/

            Ultimately, the distinction might be between what is part of the overall causal network and what isn’t. (Abstract objects as causally inert would be outside of it.) Of course, an interactionist dualist would object, but then interactionist dualists have to contend with the fact that, as Sean Carroll pointed out, we have a complete understanding of everyday physics, and it doesn’t appear to leave room for interactionism. https://www.preposterousuniverse.com/blog/2010/09/29/seriously-the-laws-underlying-the-physics-of-everyday-life-really-are-completely-understood/

            I agree that physics eventually seems to reduce to quantum fields and their interactions, which in and of itself, doesn’t seem very physical. The universe could be structure all the way down.

            Like

          7. Causality is a can of worms in itself,

            Wikipedia:

            “Causality is an abstraction that indicates how the world progresses, so basic a concept that it is more apt as an explanation of other concepts of progression than as something to be explained by others more basic. The concept is like those of agency and efficacy. For this reason, a leap of intuition may be needed to grasp it. Accordingly, causality is implicit in the logic and structure of ordinary language.”

            If you also try to make an argument that the non-physical (e.g. mental) cannot be causative, then defining physical as what is part of a causative network is circular. It also creates a dilemma for explaining what a photon is, for example, that is traveling through space and hasn’t interacted with any other matter, that is- hasn’t caused anything. And how does the physical cause the non-physical (e.g. mental) if the mental is not part of a causative network, in which case it would be physical and, hence, able to be causative?

            Like

          8. It is a fact that we never actually observe causation. We only infer it. It’s always a theory, albeit a fundamental one.

            I personally find no reason to think the mental is non-physical, so the dilemma you describe for it doesn’t concern me. (Dualists have a number of models for how they handle it, such as epiphenomenalism, parallelism, and interactionism. The last is the most popular, and the one most vulnerable to your dilemma.)

            On the photon, it was caused, and if it hits something, it will have causal effects. Photons are interesting though because, traveling at the speed of light, they are timeless. It’s almost like they are connections across spacetime between their cause and their effect, a connection everything else experiences as the photon.

            Like

          9. “I personally find no reason to think the mental is non-physical, so the dilemma you describe for it doesn’t concern me.”

            That would make you a monist. So what would be the difference between that and idealist? If everything is one thing, whether we call it mind or matter shouldn’t matter? My argument would be both positions are the same, perhaps highlighting different aspects of reality but not substantively different.

            If the photon was caused in my scenario, it was caused near the creation of the universe, shortly (maybe some thousands of years) after the Big Bang, then the causative network involves effectively the entire universe and mind would be a part of it.

            If mind is physical, then it can be causative.

            Like

          10. “So what would be the difference between that and idealist?”

            It comes down to which theories of reality you think are most accurate. If the big bang, geology, paleontology, and evolution are accurate, then consciousness, at least in our part of the universe, is a relative late comer, arising only in the last 4% of the universe’s history.

            Idealism posits that consciousness came first. If so, then those theories are all wrong. But those theories are highly predictive of observations. Of course, you can amend idealism all you want to account for those theories, but the amount of intellectual energy required seems much higher than just accepting mind independent reality, aka physicalism.

            Or you could go for panpsychism and just assert that consciousness was around the whole time. But then physicalism and idealism do start to blur together.

            “If the photon was caused in my scenario, it was caused near the creation of the universe, shortly (maybe some thousands of years) after the Big Bang,”

            The earliest photons should come from recombination about 379,000 years after the big bang. We know those photons as the cosmic microwave background. Interestingly, what started out as a full spectrum flash has been stretched by the expansion of the universe into the microwave range.

            “If mind is physical, then it can be causative.”

            I agree, but I wonder if you’re thinking this applies to abstract objects. From the SEP platonism article (emphasis added):

            Platonism is the view that there exist such things as abstract objects — where an abstract object is an object that does not exist in space or time and which is therefore entirely non-physical and non-mental.

            https://plato.stanford.edu/entries/platonism/

            Like

        2. There you go again invoking time (or Time). 🙂

          If time is an abstract mental construction, then any notion of first would come from mind.

          Julian Barbour, The End of Time, has an interesting take on causality:

          “Most physicists have a deeply rooted notion of causality: explanations for the present must be sought in the past (vertical causality, as I have called it). This instinctive approach will be flawed if the very concept of past is suspect. If quantum cosmology really is timeless, our notion of causality may have to be changed radically. We cannot look to the past to explain what is around us. The here and now arises not from the past but from the totality of things (horizontal causality).”

          Regarding mind being causative, I was thinking more of the upward/downward causation discussion that permeated some of the Sean Carroll post and comments.

          Like

          1. Can’t recall if I’ve read any of Barbour’s stuff, but most discussions I’ve seen on why time doesn’t exist didn’t seem very convincing. Now, I could conceive of it being emergent from something else. But to say it doesn’t exist at all strikes me as a proposition that would require a lot of intellectual energy to maintain.

            Upward and downward causation is a complex topic. Did the storm destroy the house? Or was it the specific wind that reached the house? Or maybe the tree that it caused to fall on the house. In casual conversation, all could be described as “causes”. But in terms of actual physics, the tree, and perhaps the wind are things to be included in a model. Including the storm would only make the model unreliable.

            Like

          2. Just weigh in, I consider time as a fundamental property of reality. Nothing happens without a “t_0” and a “t_1” already existing for it to happen in. The idea that time emerges from anything seems incoherent to me.

            (But maybe I’ll change my mind after I finish Rovelli’s book. 🙂 )

            Like

          3. Time is perfectly fine as a mathematical construct but the question is does it exist beyond that – does it map to something actually in reality – or is it a creation of mind.

            Like

          4. I know that’s what you are saying. This is similar to the discussion we had about perceptions not needing to reflect faithfully reality.

            I’m still not sure I understand all of Barbour. I think it is similar to the block universe concept.

            I just recently happened to find that quote on causality and that may be really key to it.

            What follows is only my thoughts on it and could be vastly misrepresenting Barbour who (let me repeat) I m still not sure I understand.

            The fact that things happen one after another which we perceive as time doesn’t mean time exists in a fundamental way. The question is how do we know the past and how is the present state of the universe determined. We know the past only by records of the past that exist in the present – memories, recordings, artifacts that remain to the present. In some cases, we still infer an artifact of the past. A clock is ticking, we assume it marked 1 o’clock before it is marking 2 o’clock which is now. The records, inferred or otherwise, allow us to reconstruct a past. But fundamentally that past only exists as previous states of the universe. What exists now is only it’s present state. So does the past state determine the present state? It might seem so because as the states that represent the universe change, many remain the same, or similar enough to previous states that we seem to see a continuity that we describe as time. However, an alternative explanation is “here and now arises not from the past but from the totality of things” and, if that is the case, then time would not be fundamental. We would live in a universe that is just cycling through states that provides an appearance of time.

            Like

          5. I’m aware of other takes on the nature of time, but I’ve found them uncompelling. (I’m halfway through Rovelli’s book, and it seems like a lot of hand-waving to me. I have yet to find a compelling argument.)

            “We would live in a universe that is just cycling through states that provides an appearance of time.”

            How could it be cycling through anything if time didn’t exist for it to, first, be in one state and, then, in another state? The concept of “cycling” seems incoherent without first having a notion of time.

            Kant considered it one of our most fundamental intuitions, and I’ve always suspected that derives from time being fundamental to reality (and hence our thought process).

            Like

          6. “Kant considered it one of our most fundamental intuitions, and I’ve always suspected that derives from time being fundamental to reality (and hence our thought process).”

            Using the underlying architecture of geometry as an example, Kant demonstrates that time and space cannot thus be regarded as existing in themselves. They are synthetic a priori forms of sensible intuition. Kant’s thesis concerning the transcendental ideality of space and time limits appearances to the forms of sensibility—indeed, they form the limits within which these appearances can count as sensible; and it necessarily implies that the “thing-in-itself” is neither limited by them nor can it take the form of an appearance within us apart from the bounds of sensibility (A48-49/B66).

            Like

          7. Prior to Kant, it was thought that all a priori knowledge must be analytic. Kant, however, argues that our knowledge of mathematics, of the first principles of natural science, and of metaphysics, is both a priori and synthetic. The peculiar nature of this knowledge, Kant argues, cries out for explanation. The central problem of the Critique* is therefore to answer the question: “How are synthetic a priori judgements possible?” It is a “matter of life and death” to metaphysics and to human reason, Kant argues, that the grounds of this kind of knowledge be explained. While we are prohibited from absolute knowledge of the thing-in-itself, we can impute to it a cause beyond ourselves as a source of representations within us.

            Additional anecdotes:
            1. Synthetic a priori judgements are possible because they are grounded in the linear system of reasoning, a feature of Reality, whereas rationality as a discrete, binary system is a structured system of thought, a feature of Appearances.
            2. Kant’s view of space and time rejects both the space and time of Aristotelian physics and the space and time of Newtonian physics.

            _____
            Wikipedia

            Like

          8. Yes, I know, but Kant was wrong about geometry and math. They are analytic, not synthetic. Kant simply couldn’t have known about math understandings that occurred after him. As with Newton, he got a little left behind as our understanding grew.

            Consider the canonical example of analytic a priori: “All bachelors are unmarried.” Also consider the supposedly canonical example of synthetic a priori: 12=7+5. (Supposedly synthetic because “12” is not contained in “7+5”.)

            But look more closely at the first phrase. Make it more precise: “All bachelors are unmarried men.” Then make it symbolic: “B=U+M” Now it’s essentially identical to 12=7+5.

            Likewise, given the same understanding of maths as used in understanding the terms “bachelor,” “unmarried,” and “men,” it’s obvious the statements are identical. And obvious both are analytic.

            Non-Euclidean geometries further showed that geometry is analytic. It is true within itself (analytic) and has no necessary connection with reality which synthetic propositions must have.

            Given a modern math view, the analytic nature of math seems as obvious today as his “Copernican Revolution” idea about our internal models of reality does. Kant just didn’t have access to that view.

            Like

          9. Your examples are clearly analytical, no question on that front. Nevertheless, your examples do not address the distinction dividing analytical a priori from synthetic a priori, nor does it explain why they are different. That distinction makes all of the difference.

            Kant* claims to have discovered another attribute of propositions: the distinction between “analytic” and “synthetic” judgments. According to Kant, a proposition is analytic if the content of the predicate-concept of the proposition is already contained within the subject-concept of that proposition. For example, Kant considers the proposition “All bodies are extended” analytic, since the predicate-concept (‘extended’) is already contained within—or “thought in”—the subject-concept of the sentence (‘body’). The distinctive character of analytic judgements was therefore that they can be known to be true simply by an analysis of the concepts contained in them; they are true by definition. In synthetic propositions, on the other hand, the predicate-concept is not already contained within the subject-concept. For example, Kant considers the proposition “All bodies are heavy” synthetic, since the concept ‘body’ does not already contain within it the concept ‘weight’. Synthetic judgments therefore add something to a concept, whereas analytic judgments only explain what is already contained in the concept.

            “Non-Euclidean geometries further showed that geometry is analytic. It is true within itself (analytic) and has no necessary connection with reality which synthetic propositions must have.”

            Synthetic propositions have absolutely no connection with reality. Just the opposite is true, synthetic propositions postulate a “condition on the possibility of reality”, and it is that possibility which adds something to a concept. Does not algebra, trigonometry and calculus rely upon conditions on a possibility in order to solve for an unknown?

            _____
            Wikipedia

            Like

          10. We’re on a tangent of a tangent here, but we’re basically saying the same thing. To me, “All bodies are heavy,” has a contingency external to the proposition, which is my litmus test for synthetic.

            A key property of any useful math is that it is consistent (in the Gödealean sense). That consistency is what makes all math analytic.

            Like

          11. “A key property of any useful math is that it is consistent (in the Gödelean sense). That consistency is what makes all math analytic.”

            And the ability to achieve consistency in any form of analysis, be it mathematics, the first principles of natural science or metaphysics, one must have a foundation. And that foundation is underwritten by two dynamics. Number one: analytical a priori, the actual conditions on the ground to be analyzed and number two: synthetic a priori, conditions of the possibility of the actual conditions on the ground to be analyzed. Without synthetic a priori as an underlying qualitative property of math, we would not have algebra, trigonometry or calculus.

            Just an FYI: Analytical a priori and synthetic a priori working together in mathematics, the first principles of the natural sciences and metaphysics has never really been a dispute amongst scholars.

            Like

          12. “How could it be cycling through anything if time didn’t exist for it to, first, be in one state and, then, in another state? The concept of “cycling” seems incoherent without first having a notion of time.”

            Yes, things change, sometimes in a regular fashion. That doesn’t mean we need a container for things to change in or some fundamental property to reality for changes to happen.

            Mach: “It is utterly beyond our power to measure the changes of things by time. Quite the contrary, time is an abstraction at which we arrive through the changes of things.”

            If I understand Barbour right, we also don’t need the concept of space for objects to exist in or move in. All that exists are objects and relationships between them.

            Like

          13. So, a couple of questions come to mind. How does this account for our perceptions of space and time? Or their use in so many predictive theories?

            And does this view solve any pressing problems or paradoxes in physics?

            Like

          14. I’ll repeat part of the Mach quote. “time is an abstraction at which we arrive through the changes of things.”

            It is a question of whether it is really out there in the world or an abstraction.

            The notion that there is a container (space-time) of some sort that things exist in and happen in actually seems like the more far-fetched and unnecessary idea. What is it made of? Is it physical? Can we measure it? Is it a substance?

            Barbour in some ways is a sort of re imagining of late 19th and early 20th century physics. He goes back particularly to Mach and Dirac. Mach himself was a huge influence on Einstein :

            “When Newton created dynamics, he claimed that the phenomena of the universe, especially inertial motion, unfold in an infinite invisible absolute space. In the 19th century, Ernst Mach argued that all motion is relative and advanced the revolutionary idea that inertia does not arise from the guiding effect of absolute space but from the dynamical effect of the entire universe. This idea, now known as Mach’s Principle, was the biggest single stimulus to Einstein’s creation of his general theory of relativity. ”

            Of course, Mach got some things wrong. He didn’t believe in atoms, for example. But he was mainly working in the 19th century and I think even Einstein, who was his pupil, thought he might have some up with the theory of relativity if he had known earlier in his career that the speed of light was constant. So there is huge compatibility between Einstein, Mach, and Barbour.

            The main problem that Barbour is trying to address is the unification of quantum mechanics and relativity in cosmology.

            Like

          15. “It is a question of whether it is really out there in the world or an abstraction.”

            How could it be an abstraction if there is no time in which to form such?

            Abstractions, after all, are just simplified descriptions. They are based on observation and reduction… all of which seem to require time and intelligent processing. How can you even create an abstraction — an intellectual exercise — if time doesn’t exist in the first place?

            (If change is considered simply as an ordered set of states, what are those states ordered by? Or is the claim there is no ordering?)

            Like

          16. One more note to this:

            “So, a couple of questions come to mind. How does this account for our perceptions of space and time? ”

            Space and time are real but real in the same sense abstractions are real. They are not physical.

            All of our measurements of space-time are based in relation to measurement of objects and events. We do not have a way of independently measuring either space or time apart from the relationships of objects or in reference to them. We see this most clearly in the origins of measurements. Time from rotation of earth, orbit of moon around earth, orbit of earth around sun, and (in Mayan astronomy) orbit of Venus. Distance from the length of the foot or a stride.

            Although Barbour is definitely physicalist, I see a correspondence to Hoffman regarding perceptions:

            “If we can assume no simple (e.g., isomorphic or homomorphic) correspondence between our representations and the objective world, how can we explain our successful interactions with the world? We will flesh out an answer to this question in formal terms below, but in short the answer is that this is possible, because we do not simply passively view the world, but also act on it, and moreover we perceive the consequences of those actions. In other words, it is possible to interact with a fundamentally unknown world if (1) there are stable perceptual channels; (2) there is a regularity in the consequences of our actions in the objective world; and (3) these perceptions and actions are coherently linked.”

            Like

          17. So, the distance to various objects matters. It has causal consequences. If the distance of a predator is greater, it has to undergo more change in order to be a threat than if it’s closer. If food is closer, then less change is required to reach it. Change and distances seem to have dimensions of freedom, three of which seem optional and reversible, and one that doesn’t. These relations don’t seem at all abstract to me, at least in the sense of being causally inert.

            Now, you can insist that time is something separate and apart from the quantity of change, that space is something separate and apart from distances, along with being separate from those dimensions of freedom. If so, then maybe they are abstractions, but only because we’ve redefined them as such.

            It seems to me that a much better approach would be to argue that they’re emergent from something. Of course, that requires identifying what they emerge from.

            And if we’re going to regard space and time as abstractions, then why not objects? Or each other? If all of this is an illusion, then the illusion arguably is the reality, and we have little choice but to play the game.

            Liked by 1 person

          18. Would you call “numbers, mathematical relations, properties such as redness, structures, patterns, etc. emergent also?

            What do they emerge from?

            Like

          19. With the exception of “redness” the others “emerge” from intelligent analysis of reality, but that’s probably not the question you’re asking.

            Like

          20. “How could it be an abstraction if there is no time in which to form such?”

            We have records of the “past” in the present – memories and physical recordings like the geography of the earth. Barbour calls them time capsules and notes his choice of term as somewhat unfortunate with its usual connotations. A time capsule is “any fixed pattern that creates or encodes the appearance of motion, change, or history.”

            This is where it gets interesting, although still not following him all the way. The configuration space of the universe will have places where there a lot of time capsules and other places where there are few. The places where there are a lot of time capsules are the places where there are more likely to be conscious observers.

            Like

          21. “Redness might have emerged from intelligent analysis of reality…”

            Sure. I was just distinguishing it from the other items, which are abstractions.

            “We have records of the ‘past’ in the present…”

            So at a given moment, information is presented that your mind will analyze to produce an abstraction? (Without time existing in the first place.)

            Or are you comparing the abstraction of time to the abstraction of, say, a circle, which seems to have a separate existence we discover rather than invent?

            Like

          22. “(If change is considered simply as an ordered set of states, what are those states ordered by? Or is the claim there is no ordering?)”

            Interesting you should bring that up. Unfortunately the explanation is complicated and I am not sure if I can explain it. He has a whole chapter on toy universes built of two groups of triangles, one labelled Current Theory, the other Timeless Theory.

            As you observe, the Current Theory triangles can be ordered. His objective is to show how the appearance of ordering can come from the Timeless Theory ones which are an enormously large set of triangles with duplicates that reflects all possible triangles and effectively cannot be ordered.

            Like

          23. “He has a whole chapter on toy universes…”

            The problem I have is that one can prove almost anything with the right toy universe.

            I mentioned I’m reading Rovelli’s The Order of Time, and his view is similar to things you’ve mentioned. I’m just not compelled by the arguments because, to my eyes, time always sneaks in the back door. These arguments usually can be reframed to see time as the background (and some of those arguments seem to bend over backwards to exclude it).

            Rovelli (who works on loop quantum gravity) has math without a time variable, but when he talks about the system being in state S and then evolving to state S’, it sure sounds like time is involved. Removing the variable t and replacing it with some other operator just seems to try to hide good old t under the need to look at the system twice to see both states.

            FWIW, Rovelli seems to hold time emerges from very low-level processes (spin foams), and I find myself wondering if that all but amounts to time being fundamental if it arises from the Planck level. But I still wonder how a spin foam can change state if time doesn’t exist in the first place. 🤷‍♂️

            Like

  8. First I’d like to say that I really enjoy reading your thoughts. I personally look for the deeper truth in the world around us and often am not pleased with how modern science is basically an extended version of ancient humanism. Meaning that humans are somehow removed from the natural order like saying what humans have done is not part of nature. We have built nearly all of our science on this concept and it is completely frustrating to me. Although this is not exactly what your seeking it is, in my mind, along the same path.

    I believe that fundamentally thoughts do exist in the world physically. Chemical reactions occur, pheromones are emitted and polarity changes occur when we use our brains. While on paper those may be minuscule they still exist in the world physically. I also believe that everything that exists, does so as part of an efficient cycle. I will take this a step farther and hope that you all can follow me.

    There are times when a mother knows that something has happened to one of her children when that child was far away. Most of you are likely scoffing because you think I am somehow talking about magic. Right? Be that as it may, the occurrence of knowing what physics says can’t be known does occur. But instead of dismissing this as fantasy, it is actually more realistic to understand this as part of the natural world. The natural wold has an energy transfer system that supports this occurrence. I know a person who would not lie experience knowing what should not be knowable. There is a path of common sense that you can follow to this conclusion. It requires the ability to look at how things are happening in front of you without prejudices. Ahh I’m getting preachy I apologize. Physics will unlock this energy system one day I am sure of that.

    Like

    1. Thanks Darren.

      On a mother knowing things about her child, I think we have to be cognizant that often we know things through unconscious knowledge, with only the result surfacing into consciousness. This may be completely mechanistic, but the result, because we’re not conscious of the underlying information transfers, seem miraculous and magical. In other words, a lot of times we have the impression of clairvoyance without the reality.

      Do we ever have clairvoyance itself? I’m afraid that’s one of the things that don’t stand up to the reproducible or otherwise verifiable criteria. For me to believe it, someone will have to be able to demonstrate it repeatably and reliably, or demonstrate a high and wide enough occurrence that we can consider it reliably verified. I used to read about these kinds of things, but I never saw this level of repeatability or verification.

      Saying a hundred people one hundred years ago saw it doesn’t count, but producing 100 independent unbiased witnesses who can each give their own testimony under rigorous scrutiny, might. Although being able to reproduce it under laboratory conditions would be better.

      Like

  9. I basically follow Quine on this, Is there a number greater than 3? Yes, so there is a number. “There is a ” is the litmus test for “Do X’s exist?”, so, numbers exist. On a neoplatonist view where we abolish the “separate realm” which Plato supposed was “where” numbers exist, and just say that numbers exist, it doesn’t seem very earth-shattering.

    At some point you have to stop questioning your language, and just go ahead and use it. Otherwise you never get to say anything.

    Like

    1. Yeah, the whole “realm” thing probably cause more confusion than it’s worth. Interpreted with charity, it just means we won’t find them out in the world.

      As I noted in the post. I’m uneasy with the language argument. I’m a little bit easier with the argument that they’re a vital part of our theories.

      Like

    2. Paul, that formulation seems problematic to me, because I think everything ends up existing in one context or another. Numbers only exist in the context of rules which say they exist. For example, does the square root of -1 exist? Not in the context of math that gives us standard high school algebra. However, in the context of imaginary numbers, and I believe quantum mechanics, the answer seems to be yes.

      Is why I prefer to say that physical stuff exists, patterns (like numbers) are real, and some patterns (like numbers) are discernible in physical stuff while other patterns (“married bachelor”) are not so discernible.

      *

      Like

      1. The mathematician in me can’t resist…

        “For example, does the square root of -1 exist? Not in the context of math that gives us standard high school algebra.”

        I learned it in high school. It pops out from the fundamental algebra rule about polynomials having at least one root.

        Consider the simple polynomial: x^2 + 4

        The only way for it to have a root is for X^2 to equal -4, and the only way that can happen is with i. Thus the roots for the equation are +2i and -2i.

        The square root of minus one turns out to be necessary!

        Like

  10. You might even be a realist, that is focused upon what is real. Because we possess imaginations we can come up with conjectures that are either unprovable or close to it. One such is that “reality is a simulation, a la The Matrix. Another is any of the various gods.

    Platonism is real in that it is an accurate statement (apparently) of a conjecture posited by Plato. Is the conjecture valid? I don’t think so but it hardly matters as it is quite unprovable. So, we move on. The advantage of the scientific method is there is an arbiter of truth, it is called nature. Philosophy has no such arbiter (not even logic), nor does theology. These subjects basically throw things against the wall to see if they stick. Plato’s ideas have stuck arund for a long time but are basically useless (except to theologians).

    Liked by 1 person

    1. Don’t forget Steve, that this so called “arbiter of truth” referred to as nature, or the natural world is an arbitrary distinction which materialists throw against the wall; and once it sticks, scientists successfully build constructs which are useful. None of those constructs reflect the true nature of reality. If useful-ness itself is the arbiter of truth, then both philosophy and science share the Nobel prize.

      Essentially, Plato and Aristotle’s philosophies are a complete bastardization of Parmenides brilliant insights into the reality/appearance distinction. The reality/appearance distinction is the only architecture which can avoid the paradox of dualism.

      Like

    2. Emotionally I’m a scientific realist. I get excited by science as the pursuit of truth.

      But intellectually, I acknowledge that knowing truth only happens through models that successfully predict future experiences. In truth, I see the realism vs instrumentalism distinction as invalid. Your model of your house is just as much a theory as your model of the atom. Neither is guaranteed to accurately reflect reality. They are only more or less accurate in their predictions. But to a hard core realist, that makes me an anti-realist, an instrumentalist.

      And strictly speaking, as an instrumentalist, the platonism question doesn’t matter. A platonist and a nominalist are not going to have different predictions, at least not ones that can be tested. Although some platonists do make predictions about mathematical multiverses and the like.

      Like

  11. Cool post! Ever since those Chalmers posts I wrote, I’ve been thinking about abstraction and information, so this fits right in.

    “…where non-physical ones seem to struggle with it.”

    Given that empiricism and physical theories are so strongly linked, it’s not surprising non-physical theories would struggle so.

    It might be good to unpack exactly what one means by “non-physical theory” — does finding a lawful explanation make it a physical theory? What does a coherent non-physical theory amount to?

    Chalmers uses matching abstractions between otherwise different physical systems to make a claim of identity between those systems. Would that be a non-physical theory in your book?

    “Numbers, mathematical relations, properties such as redness, structures, patterns, etc.”

    Is “redness” really an abstraction? It seems to stick out from the rest.

    Certainly photons in the “red” frequency range are concrete, so I assume we mean the perception or qualia of red, but can we really assume that’s an abstraction? It’s something that really happens when we look at red. I’m not sure it belongs with the other items you listed.

    “But is platonism true?”

    In what sense “true”?

    The question doesn’t have the same ontological weight that idealism versus realism does, or that materialism versus dualism does. A platonic realm, by definition, is inaccessible to us, except through thought. It’s being true, or not true, doesn’t alter our sense of reality as much.

    Ask it this way; Is Sherlock Holmes real? (You know exactly who I mean, so it’s hard to argue he’s not real. At the same time, think how diffuse and varied the concept “Sherlock Holmes” has gotten over the years.)

    “Mathematicians seem convinced that they’re discovering something, not developing tools…”

    For me, this is the heart of the question. Is a circle a discovery or an invention? Was the concept always “there” (“somewhere”) to be found? Would it be found by any intelligence? Would it necessarily be found by all intelligences?

    If so, it’s hard to argue it doesn’t have some kind of separate reality on its own.

    “A ruthless application of Occam’s razor seems to make it disappear in a flash of parsimony.”

    (Of course, nature isn’t required to obey heuristics. Occam isn’t law.)

    This gets back to what we mean saying platonism is true. Abstractions are reductions. They are also descriptions. In saying platonism is true, all we’re really saying is that various physical instances can reduce to a common description.

    The interesting thing to me is that these common descriptions exist (whether invented or discovered), that basic patterns and laws lead to such interesting complexity.

    Liked by 1 person

    1. Thanks!

      “It might be good to unpack exactly what one means by “non-physical theory” — does finding a lawful explanation make it a physical theory?”

      I actually did a post a while back asking what, fundamentally, we mean by “physical”. I linked to it at the beginning of the post. In many ways, the physical is what passes that empiricism test. If we think of empiricism as physical measurement, this can seem like we’ve locked ourselves into a closed loop. That’s why I think it’s important to remember that empricism is, ultimately, about conscious experience. We just use measurement to refine what that experience is telling us.

      But it does make your question about what a coherent non-physical theory would look like an important one. If we developed such a theory, and it was predictive, would we regard it as non-physical, or as expanding physics? In that older post, I mentioned dark matter, dark energy, and originally even gravitation, as things that we ultimately just regarded as expansions of physics rather than supernatural. (Some of the natural philosophers in Newton’s time weren’t so sure about gravitation.)

      I wondered if anyone would say anything about “redness”. It does get used in discussions about subjective experience, as a phenomenal property. It appears to be a relation between red and our perception of it. But red itself appears to be a relation between whatever reflects long wavelength light and our perception of it.

      Which raises the question, what about the relation between redness and our perception of it? Rednessness? And what about the relations with it: rednessnessness? We can find ourselves spiraling down an infinite regress, particularly if every one of these properties are held to actually exist.

      It’s worth noting that the SEP article on platonism mentions that it’s not an all or nothing proposition. One can be a platonist about some things and not others. A mathematical platonist isn’t necessarily a property platonist, etc.

      “Is Sherlock Holmes real?”

      The character, or as you note, the collection of characters, or perhaps the category of character, definitely seems to be real. But if in centuries to come, the character is forgotten and any work with him lost, will he still exist? Do the forgotten gods of prehistoric societies, as concepts, still exist?

      “Would it necessarily be found by all intelligences?”

      That seems like a key factor. It’s hard to imagine an extraterrestrial intelligence wouldn’t have the same mathematics we have. Certainly radically different notation, but the underlying structure seems like it would have to be there.

      “Of course, nature isn’t required to obey heuristics. Occam isn’t law.”

      Definitely. Occam seems to derive it’s power from the fact that each speculative concept has a chance of being wrong, so a theory with lots of unnecessary ones seems to have a much higher chance of being wrong than one that keeps them to a minimum. But it’s definitely a probability thing, not a hard and fast law. Still, it appears to be a very effective heuristic.

      “In saying platonism is true, all we’re really saying is that various physical instances can reduce to a common description.”

      The question is whether the description has an existence outside of any minds. On the nominalist view, we form that description, it exists as a model in our minds, and as the physical pattern of instances, and that’s it. The platonist considers the description, in and of itself, to have an existence.

      Like

      1. Regarding “redness”, would it change your (Wyrd’s or Mike’s) analysis if you change the topic to “whiteness”? Obviously there are no white photons, yet we have no difficulty attributing the property of whiteness to a ball, and that attribution seems to be approximately the same kind of attribution as the attribution of redness.

        *

        Like

        1. I can’t speak for Mike, but from my perspective qualia of any kind aren’t very relevant to platonism. They aren’t, themselves, any kind of abstraction in my eyes, even if they’re ultimately non-physical. (Just like ghosts wouldn’t be abstractions if they were real.)

          For me, platonism is mainly about the reality of ideas (i.e. abstractions).

          Like

        2. It doesn’t really change my analysis here except for the fact that the initial white relation involves move wavelengths.

          Interestingly, white (or more accurately grey scale) is what we get from rod cells. In essence, when all cone types are equally stimulated, it gets translated into the same signal as what the rods send. That feels like it makes white less arbitrary.

          Like

      2. “That’s why I think it’s important to remember that empricism is, ultimately, about conscious experience.”

        This elevates consciousness, and I find that key. How are you on equating consciousness with quantum measurement — that consciousness is the only thing that causes true collapse?

        “…things that we ultimately just regarded as expansions of physics rather than supernatural.”

        Good way to put it, that’s kind of what I was asking. To what extent would you tend to equate non-physical theories with the supernatural, almost by definition?

        (Chalmers seems to stride this divide with one foot in naturalism (aka physicalism as in contrast to materialism) and another in psychophysical (non-physical) principles. But isn’t any lawful behavior we find physical?)

        “I wondered if anyone would say anything about ‘redness’.”

        It kind of jumped out at me. I think we’re actually discussing two separate topics here. Firstly, about the reality and types of abstractions and ideas. Secondly, whatever application that may have to consciousness. (Perceptions of red clearly in the latter category. I’m more focused right now on the former.)

        “But if in centuries to come, the character is forgotten and any work with him lost, will he still exist?”

        You hit on an important point: what (if any) is the connection between existing and time-span? You and I only exist for a brief time. Our existence has a definite beginning. It will taper off as all evidence of us fades and vanishes — like many fictional characters already have, like Sherlock Holmes might. (But one might like to imagine that idea is immortal.)

        Compare with circles (or math in general), which seems to have existed prior and somehow outside of any time or place. Sherlock Holmes might be unique to Earth. Math is universal.

        (Can we make anything of that Holmes could use math, but math doesn’t use Holmes? 🙂 More seriously, there does seem an ontological hierarchy here.)

        “The question is whether the description has an existence outside of any minds.”

        At least in some cases, it seems the answer is yes. But it also seems minds are a key aspect in realizing them.

        I think this is because only minds create abstractions (descriptions, ideas). Without intelligence, reality is entirely physical (this seems clear). But intelligence seems to change the equation — it’s not just a new aspect of reality, it’s an aspect that can describe, predict, and create.

        (Which is exactly why I’ve said it’s “special.”)

        Like

        1. “How are you on equating consciousness with quantum measurement ”

          It seems like if it were true, then quantum computing would be much easier to engineer. All that would be necessary was ensuring that no conscious agent had access to what was happening. But read about it, and decoherence comes across as the bane of their existence. Interaction with the environment seems to be the main thing. Of course, that gets into what “the environment” is. Arguably for a subatomic particle, its “environment” is the atom and molecule it’s in, yet large molecules have been kept in superposition.

          Sean Carroll seems to keep smiling at us from the many-worlds corner wondering when we’ll give up and walk over.

          “To what extent would you tend to equate non-physical theories with the supernatural, almost by definition?”

          Lately I’ve tried to avoid the word “supernatural” because of its connotations, preferring words like super-physical. But I have to admit that in my mind, it’s the same thing.

          “Chalmers seems to stride this divide with one foot in naturalism (aka physicalism as in contrast to materialism) and another in psychophysical (non-physical) principles.”

          Since reading Chalmers’ paper, I’ve come across the term “psychophysical” a lot, or more likely I’m just noticing it more. The majority of its usage appears to be about the mapping between the psychological and the physical in a theory neutral manner, agnostic as to whether the mapping is between the physical and non-physical or between the physical at one layer of abstraction and the physical at another.

          “But isn’t any lawful behavior we find physical?”

          That feels true. It also makes me realize why so many spiritualists are fascinated by quantum physics and want to work it into consciousness, because of its unpredictable nature. Ultimately the supernatural is about defying prediction, breaking out of the regularities that science seems to have so relentlessly consigned us to.

          “You and I only exist for a brief time.”

          Indeed. And after we’re dead, we effectively become concepts, not that different from the concepts that never had physical form. Alexander the Great and Achilles are effectively both characters for me. It’s just that Alexander’s name has a reality asterisk by it.

          “I think this is because only minds create abstractions”

          A platonist would insist that we discover rather than create them. But if it’s true that only minds create them, then nominalism seems more relevant.

          Like

          1. “Sean Carroll seems to keep smiling at us from the many-worlds corner wondering when we’ll give up and walk over.”

            As an aside, I’m about 2/3 through reading Through Two Doors at Once (Anil Ananthaswamy), and he’s made the point that MWI answers a lot of puzzling questions. I have ontological issues with it (talk about proliferation of entities), but I do see its appeal.

            My question was about your connecting empiricism with conscious experience.

            “A platonist would insist that we discover rather than create them.”

            In some cases, yes (math), but in other cases it doesn’t seem so (Sherlock Holmes).

            Regardless, the overall point is that only intelligence deals with information systems. Only intelligence discovers math or invents Holmes.

            Naturally occuring physical systems all have information as a property, but only intelligent minds deal with information systems in which the information itself is the primary property of the system. Only intelligent minds create information systems that are substrate agnostic.

            Unless the brain is the one naturally occurring physical system in all of reality that is actually an information system in disguise.

            Like

          2. “My question was about your connecting empiricism with conscious experience.”

            I thought you were asking about consciousness and the wave function collapse. If not, then sorry, I missed the actual question.

            Like

          3. I was questioning the assertion “that empricism is, ultimately, about conscious experience.” I asked about quantum measurement in comparison, because it seems a similar situation.

            There is a controversial belief consciousness is the only thing that actually collapses a wavefunction. I think we agree this seems unlikely. I’m asking what is it you see in empiricism that makes it about conscious experience.

            Like

          4. Ah, okay. Sometimes I take things to be obvious that aren’t. Words like “empiricism” and “empirical” tend to get thrown around in a manner that obscures what they actually mean. There tends to be an impression that it’s a uniquely scientific thing involving complex equipment and procedures. Admittedly the way I worded my point assumed what I’m about to say was already understood.

            Fundamentally, empiricism is experience. The word comes from the literal Greek term for experience. When we say “empirical measurement” we could instead say “experential measurement”. This makes sense when you consider that the word experiment means controlled and structured experience. It all comes down to knowledge gained from sensory experience, that is, conscious experience.

            Measurement is simply putting a comparison in the experience. And measuring equipment can only be used once it’s been demonstrated to show accurate information. Galileo was criticized for reaching conclusions by looking through a telescope, but those making the criticism missed the fact that the telescope could also be used to look at faraway things on Earth, and then walk up to those things to verify what was seen.

            Like

          5. Agreed. So…

            “It all comes down to knowledge gained from sensory experience, that is, conscious experience.”

            You agree consciousness has a special role in reality?

            Would you also agree it is what recognizes and codifies abstractions?

            Like

          6. “You agree consciousness has a special role in reality?”

            No, I can’t see that. Only in our knowledge of reality. Reality seemed to go on a long time before consciousness came along, at least according to our most predictive scientific models.

            “Would you also agree it is what recognizes and codifies abstractions?”

            Sure. It’s what makes me wonder if there’s any need for the platonic versions.

            Like

          7. In regards to knowledge? Sure, but that just seems to be saying that consciousness is special to itself.

            I’m curious why you seem so hung up on this “special” thing? Yes, I said that there’s nothing special about the physics of the brain. (I don’t think anything I’ve said here contradicts that.) And I noted how much the differences between us and non-human animals mostly come down to differences in quantity rather than quality.

            But I also said that we have symbolic thought (complex language, math, art, etc), which at least for the time being, seems mostly specific to humans. If you’re looking for something to affix “special” to, can’t it go there? Or is that maybe what you were looking for with the abstractions question?

            Like

          8. “I’m curious why you seem so hung up on this ‘special’ thing?”

            I suppose I keep gnawing at the bone because I have a hard time wrapping my head around the idea that someone could deny that brains are special objects in reality based on objective grounds. While I might not always agree (due to different axioms), I usually understand your views. This one puzzles me, so I’m trying to understand it.

            “Yes, I said that there’s nothing special about the physics of the brain.”

            I know (and understand how) you believe that, and I agree it may turn out to be correct. I’m saying that, even in this context, brains seem a special creation of reality (for reasons I’ve listed many times).

            “And I noted how much the differences between us and non-human animals mostly come down to differences in quantity rather than quality.”

            We’ve also discussed the vast gap on that putative scale between human brains and all animal brains (as you go on to note). In this context, brains (all brains) are still pretty special objects given what they allow their owners to do. Even insect brains accomplish some astonishing things.

            Human brains take it so much further: we discover and invent abstractions, we deliberately explore our reality (internal and external), we create information systems, we defy natural conditions that would normally kill us (and thus inhabit the planet and potentially beyond), we create medicine and art and stories.

            It blows my mind you don’t find all this really damn special! 😮

            “If you’re looking for something to affix “special” to, can’t it [involve symbolic thought]?”

            Certainly! It being that special brain that enables symbolic thought.

            “Or is that maybe what you were looking for with the abstractions question?”

            Given that symbolic thought is abstract thought, mos def! 😀

            Like

        2. But isn’t any lawful behavior we find physical?

          So far! Or at least, arguably yes so far.

          Concepts gravitate to clusters. Whether there is one cluster or two or more (relative to human perceptual dimensions) is typically an empirical question. And typically not amenable to picking a single dimension as criterial.

          If there were ghosts and demons and magic words of magic spells, and they followed their own laws that were utterly alien to QM and relativity, then not all laws would be physical laws.

          Like

          1. Ghosts, demons, and magic are all specific examples we recognize from pre-scientific understandings. But if we encounter something we don’t understand today, are we likely to call it “magic”, or assume it’s just physics we don’t yet understand? If we encountered an unknown intelligence with unknown physics today, would we assume it was a ghost or demon, or an extraterrestrial?

            For example, scientists in the 90s could have just assumed dark matter and dark energy were magic. (I’ve encountered new age spiritualist types who actually do make that assumption.) But instead they applied what mathematics they could to it and gave it names to mark our ignorance.

            Put another way, science could be considered the pursuit of magic the works.

            Like

          2. You’re overlooking my point about clusters, which is why I picked those specific examples from our early understandings. I would also note (and this is clustering too) that the leading candidates for dark matter are governed by the same laws as our familiar electrons and photons.

            Like

          3. I think my point was that the clustering is a cultural artifact. A historian thousands of years from now may see the divisions we make between magic and science as simply one between lesser and more accurate understandings of reality.

            Liked by 1 person

          4. To some small degree, clustering is a cultural artifact. For the most part though, it is biologically driven psychology, with species-wide generality. See the research on the Sapir-Whorf hypothesis for details.

            Liked by 1 person

      3. “Is Sherlock Holmes real?”

        Just wanted to point out that my formulation makes this easy.

        Is Sherlock Holmes real? Yes
        Does Sherlock Holmes exist? No
        Are forgotten gods real? Yes
        Do they exist? No

        *

        Like

        1. James,
          I can’t really say the distinction you’re making here feels meaningful to me. It seems like one you arrived at through linguistic fiat, by semantically defining words so you can use them differently, without getting at the real question.

          But maybe I’m just not understanding it? The limitations of language are frequently a thing in these kinds of discussions.

          Liked by 1 person

          1. I agree with the linguistic fiat, but I think it is the necessary path. So my question is, can we express “the real question” in my terms? So remind me in your terms (or preferably in my terms) what is “the real question”?

            *

            Like

          2. In your terms? Okay, I’ll give it a shot.

            Are abstract concepts objectively real, that is, real independently of minds?

            Note that Merriam defines “real” as “having objective independent existence” and “exist” as “to have real being whether material or spiritual”, so to me the question above is equivalent to asking if the statement below from the SEP article on platonism is true.

            Platonism is the view that there exist such things as abstract objects — where an abstract object is an object that does not exist in space or time and which is therefore entirely non-physical and non-mental.

            https://plato.stanford.edu/entries/platonism/

            Like

          3. In my terms? Fail. 🙂

            So there are a couple terms in there which are not well explained, namely “spiritual” and “being”. I don’t know how you determine what has “being”, and “spiritual” clearly has a lot of unwanted baggage. But combining Merriam and SEP, you get platonism as the view that abstract objects are spiritual beings, and I’m not sure what that buys you. For myself, I think of a being as a physical thing. I don’t see the number 7 as a being. Maybe I’m too much influenced by Shakespeare.

            *
            [To be, or not to be. An abstraction would not have the option not to be.]

            Like

          4. Perhaps to shortcut some posts:

            You will be able to define abstractions as not real and not existing. That’s fine, then you get

            There are abstract objects which are not real and don’t exist and which are mind independent. There are abstract objects which, even though they are not real and don’t exist, can be discerned and differentiated in physical objects (by other physical objects). There are abstract objects which are not real and don’t exist but can be associated with unique physical objects that are real and do exist. There are physical objects that are real and exist whose sole function is to represent a specific abstract object which is not real and does not exist.

            In this context, “redness” and “whiteness” are not real and don’t exist, but are discernible in certain physical objects (balls) by other physical objects (brains).

            *

            Like

          5. I’m having a hard time disentangling all this. How can something not be real, not exist, and yet be mind independent?

            Sorry. Might be I’m just too tired and need to look at it again tomorrow.

            Like

          6. It’s a matter of supplying the semantics (intention) behind the words. Semantically, “real” and “exists” are the same thing — that was the point of my re-phrasing the Sherlock Holmes thing. They are synonyms.

            There are categories of meaning behind “real” — there is a sense in which the number pi, Sherlock Holmes, and my love of caramel, are all “real,” but the sense in which they are real differs.

            A big dividing line is “physically real” — a property Sherlock Holmes lacks (given we define Holmes as the character invented by A.C. Doyle). The number pi is also not physically real, but unlike Holmes, it is eternal and discoverable by any intelligence.

            That last trait makes pi (and math in general) seductively like physically real things, because they, too, are discoverable. Math (unlike Holmes) does seem “out there” to be found.

            To the extent my love of caramel is real, math and Holmes are a cinch.

            Maybe one broad definition of “real” is: Anything we can describe. (Sometimes our descriptions are fiction, sometimes fact.)

            Or perhaps simply: Anything that has information is real.

            Like

          7. For me, it comes down to whether the concept is epistemically useful. Does its existence help in understanding (predicting) the world? Or hinder it?

            The problem with abstract objects is that they seem to sit on the border. It’s not hard for me to understand the world without them, with just the physical patterns, and the mental models we construct of those patterns. But admittedly it requires more energy than just assuming those patterns in and of themselves have their own existence. It puts me in the position of having to constantly remind both myself and others that the extra entity isn’t there.

            With physical concepts, there’s also usually an epistemic cost in assuming their existence. For example, if I assume fairies exist, then the lack of evidence for them becomes an issue. But abstract objects don’t appear to have a cost. They have no spatio-temporal properties and are causally inert. We don’t have to explain the lack of evidence for them.

            Which implies assuming their existence is the most productive option. But I might feel different later.

            Like

          8. “It’s not hard for me to understand the world without them, with just the physical patterns, and the mental models we construct of those patterns.”

            But aren’t those mental models abstractions? Isn’t recognizing those patterns as similar an abstraction?

            Abstractions are just descriptions. Animals can function in the world without them, but I’m not sure intelligent beings can (unless we choose to live “in the moment” like animals).

            Can you build a house or bridge without a plan? A plan is an abstraction (description).

            “It puts me in the position of having to constantly remind both myself and others that the extra entity isn’t there.”

            If you don’t see it as “extra” in the first place, but simply as a description or idealization, then there’s nothing to fret about.

            The major fly in the ointment is mathematics and the whole “eerily effective” business. That, to my eye, is the only place this gets weird.

            Like

          9. “But aren’t those mental models abstractions?”

            Not in the platonic sense of an non-physical causally inert object.

            “Abstractions are just descriptions.”

            If that’s true, then there’s nothing preventing them from being only physical representations (either in minds or on paper).

            ” Animals can function in the world without them”

            It’s worth noting that animals with distance senses do have foresight and imagination. They’re not just reactive beings. Although they don’t have symbolic thought and the scope of their foresight is generally far smaller than what humans have.

            “The major fly in the ointment is mathematics and the whole “eerily effective” business.”

            To me, it’s not really all that eerie, regardless of whether platonism or nominalism is true. Mathematical constructs seem based on patterns in the world, as well as fictions derived from those patterns. If that’s right, then the effectiveness is just reciprocal.

            Okay, now I’m veering back to nominalism. Told you it might be different later. 🙂

            Like

          10. “Not in the platonic sense of an non-physical causally inert object.”

            We must mean different things, because from my perspective that’s clearly false. We see physical things in a certain repeating pattern of physical reality and understand them as a common abstraction — a “circle.”

            Even the word “tree” is an abstraction — there being a platonic form of tree — necessary for a human understanding of reality. We recognize trees as belonging to an abstract class or category.

            “If that’s true, then there’s nothing preventing them from being only physical representations (either in minds or on paper).”

            Of course not, but I don’t see how that interacts with abstractions being descriptions? We certainly agree information of any kind needs be physically reified to be useful. I believe we also agree information is substrate agnostic.

            All I’m doing is equating the concept of abstraction with the concept of description to highlight the intention or purpose of abstraction.

            “It’s worth noting that animals with distance senses do have foresight and imagination.”

            Hang on, I’m lost. You started by saying you don’t need abstractions, but are you now arguing that even animals do?

            “Mathematical constructs seem based on patterns in the world,”

            You’ve said you’re not that into mathematics (and I know you skip or skim my math-oriented posts). I think it’s possible that, if you got deeper into it, you might find why so many mathematicians do find it eerie.

            Something as simple as a ballistic object — a thrown rock, say — describes a parabola, essentially just ax^2 which seems sparse given it applies to so many situations.

            Or the way pi and e appear so often in natural processes. Many are freaked out by i, but it pops out of math naturally. Nature seems deeply involved with an “imaginary” constant.

            Like

          11. “We must mean different things, because from my perspective that’s clearly false.”

            Doesn’t sound like you’re a platonist then. At the moment, neither am I, so we might be on the same page.

            “You started by saying you don’t need abstractions, but are you now arguing that even animals do?”

            It seems like maybe there’s a confusion here between abstract mental content and abstract objects in the platonic sense. Again, according to platonism, these abstract objects have no spatio-temporal, causal, physical, or mental properties. I don’t doubt abstract mental content. I do doubt the platonic version. (Although if you think the answer here is obvious, it probably means you haven’t really thought it through.)

            On mathematicians, as I indicated in the post, their platonism does give me pause. And I get the feeling of awe when so much is explained by so little. Science has a lot of that too. But as far as I can tell, a scientific theory that is derived from other theories, but fails to make accurate predictions, is usually regarded as wrong, whereas a mathematical model with a similar pedigree gets the designation “abstract”. Indeed, a defense of string theory is that it’s been productive for mathematics.

            Like

          12. “Doesn’t sound like you’re a platonist then.”

            Do I believe there is a literal realm of platonic forms? No. Do I believe abstract forms (such as circles) have, in some sense, a separate and eternal form? Yes. (In particular, I think we discover math fundamentals, which is essentially a platonic view.)

            My understanding is that it’s not clear how literally Plato meant his world of ideal forms to be taken. He may have just been talking about that separate and eternal form. (He may simply have been one of the first to recognize the “eerie effectiveness” of math.)

            Platonism can be read to be nothing more than a statement that we live in a lawful reality — one with regular patterns and laws we can discover.

            “It seems like maybe there’s a confusion here between abstract mental content and abstract objects in the platonic sense.”

            What I said originally: “But aren’t those mental models abstractions?” The answer is yes.

            You’re saying you don’t see the need for a platonic realm, but I’m not sure “need” is the issue. It’s more just a recognition of the reduction involved in abstraction. It’s a deep (and ancient) question about the precise nature of existence.

            I keep coming back to circles (or the Platonic solids). It’s almost impossible to see those ideas as not having a separate existence. The idea that we “invented” them seems very hard to defend.

            “But as far as I can tell, a scientific theory that is derived from other theories, but fails to make accurate predictions, is usually regarded as wrong, whereas a mathematical model with a similar pedigree gets the designation ‘abstract’.”

            Some mathematical models are different from physical models in that (mathematical) consistency is often the only requirement. They aren’t intended to reflect our physical reality. (For example, a lot of string theory work is done in AdS space, but we live in dS space.)

            Even so, one aspect of math’s eeriness is how often abstract mathematical models turn out to have relevance in our physical world. (In some cases their authors explicitly said, ‘this will never be useful; it’s just interesting.’ Then down the road it turns out to be hugely useful.)

            Purely mathematical models fail if the math is bad. Physical mathematical models (string theory, QFT, LQG, GR, Newton, etc) fail if they’re experimentally falsified. It’s the same thing in different realms.

            Like

          13. “Do I believe there is a literal realm of platonic forms? No. Do I believe abstract forms (such as circles) have, in some sense, a separate and eternal form? Yes.”

            The first is a strawman, albeit one people seem unable to resist. By the second, I take you to be a platonist (again, small ‘p’).

            As I understand it, Plato is widely regarded as being pretty strong in his conception of Forms, which is why when referring to his concept of them, they’re usually capitalized. Aristotle pulled back from Forms to forms. But Aristotle wouldn’t count even as a small ‘p’ platonist. He appears to be been what the SEP article calls an immanent realist.

            “You’re saying you don’t see the need for a platonic realm, but I’m not sure “need” is the issue.”

            By “need” I meant epistemic need. Since most speculative concepts are wrong, I’m generally going to be skeptical of a conception unless there is evidence or epistemic need for it. But as I said above, platonic objects seem right on the threshold. I’m starting to wonder if Chalmers is right about there not being a fact of the matter here.

            “It’s almost impossible to see those ideas as not having a separate existence. The idea that we “invented” them seems very hard to defend.”

            The nominalist response is that the physical patterns exist, emerging from the laws of nature, and multiple people seeing them will independently come up with the same idealized mental models. No additional entities required.

            “Even so, one aspect of math’s eeriness is how often abstract mathematical models turn out to have relevance in our physical world. ”

            The same thing sometimes happens in physics, where it’s sometimes said, “Don’t worry. It’s not like this crazy thing is actually true, it’s just a mathematical convenience.” Max Planck said something like that when he came up with the quanta concept. And people initially said something like it for Copernicus’ model of the solar system. Mathematical convenience often presages reality. Of course, if it’s all based on reality, then nothing spooky is happening, it’s just a feedback loop.

            “It’s the same thing in different realms.”

            In both realms, an a priori failure is a failure. But in science, the theory also has an a posteriori requirement that it can fail. No one necessarily requires a posteriori success from a mathematical model.

            Like

          14. “[The idea of a literal realm of forms] is a strawman, albeit one people seem unable to resist.”

            Okay, but then you say:

            “Plato is widely regarded as being pretty strong in his conception of Forms”

            Was Plato one who was unable to resist?

            To be honest, I’m not entirely sure why we’re even discussing platonism, which, as you asked, I do think (at least somewhat that) “the whole thing [is] just hopeless navel-gazing” — a philosophical fine point of no real value.

            I’m curious exactly how you’re tying it all into consciousness.

            “Of course, if it’s all based on reality, then nothing spooky is happening, it’s just a feedback loop.”

            The thing about math is that, other than math itself being connected with reality, often these mathematical abstractions aren’t based on anything real. It’s only down the road that some weird abstraction turns out to have physical application.

            “No one necessarily requires a posteriori success from a mathematical model.”

            Right. Because math is essentially entirely a priori. All it requires is consistency.

            Like

          15. “Was Plato one who was unable to resist?”

            Modern platonism is not necessarily the philosophy of Plato. That’s why it’s usually not capitalized.

            “To be honest, I’m not entirely sure why we’re even discussing platonism,”

            Like any topic I post on, I found it interesting.

            “I’m curious exactly how you’re tying it all into consciousness.”

            For me personally, it has no bearing on consciousness. I initially thought Chalmers’ conception of non-physical experience might work as a platonic property, but as I noted in the post, Chalmers himself nixed that. It’s conceivable it might still describe the views of some property dualists and/or panpsychists, but it’s probably far too limited for what most of them envision.

            Like

          16. Oh, okay. The second paragraph of your post, and the mention of “redness,” gave me the impression you saw some connection.

            Just on topic, then, yeah, I’m a small “p” platonist (more than not), but don’t see the question as hugely significant. As you say, an interesting topic to ponder on occasion.

            Liked by 1 person

          17. You guys give up too easily. 🙂

            There’s a significant connection, in my view, between platonism and Consciousness. Specifically, a necessary property of a consciousness-type process is its association with a semiotic object, an abstraction. That object is a non-physical thing that can be (potentially) identified, described, and discussed, but may or may not “exist” or be “real”, depending on how you define those terms.

            *

            Like

          18. The semiotic relation seems true under both platonism and nominalism. But only under platonism would there be a non-physical aspect. Of course, the non-physical aspect is non-spatial, non-temporal, non-causal, and non-mental, so the end result for operations seem nil.

            Like

          1. That’s fine, as long as you are consistent. Is Julius Caesar real? No, he’s dead. Does Julius Caesar exist. Yes, as seen in lots of books and movies.

            *

            Like

          2. There is a difference in that Caesar was real person (who died), plus he also exists as a character in books in movies. Holmes was never a real person.

            Like

  12. Great post. I tend to agree with you. I know you hate when I bring up Sam Harris, but your delineation between what is useful and true reminds me of the “great” first podcast he did with Jordan Peterson where Peterson argued that only useful things were true, and Sam Harris simply couldn’t understand how one could define truth in that way, and they just got stuck on that for an hour and half. lol I suspect Jordan Peterson would lean towards platonism, Sam Harris would not.

    A lot of my philosophical journey into religion and the brain was initially spurred by a Douglas Adams speech that was transcribed in his last posthumous publication “The Salmon of Doubt”. The speech is called “Is There and Artificial God?” Worth a read as it’s online as well. Anyway he talks about situations where science has tried to come along and figure things out, but models created by religion work better. Now it’s likely that what has happened is that the time history of observations by a culture has locked on to a pattern that modern science isn’t seeing, but to me it was a good sort of illustration between useful things and truth. What it really means though is that we haven’t really figured out what’s true yet, so let’s go with what is useful for now. So while we might have discovered something useful it doesn’t mean we have an accurate description of reality, for humans having useful things is typically more important to us. As one said above that science can be an arbiter of this, and while we are using the useful thing, we can still work on finding out how it really works. What example in my field is with tornadoes. Modeling a tornado is difficult since we don’t understand all the conditions necessary for their formation, but we have many different approximations that are useful. If one of those approximate models can reliable save a few lives (even if not every life) because we can improve warning times reliably by an average of 3 minutes, we are going to go with that model, even while knowing it can be better.

    Patterns and numbers are useful. We know about them because we are pattern seekers, and if we see a pattern we are going to try and describe it. Once we have a pattern we can remove the pattern from the phenomena and see all sorts of possibilities about where it might lead. If I find something property that decays exponentially, once I have that pattern, I can manipulate numbers and create all sorts of functions. Now I have new patterns, and as a pattern seeker I can see if other phenomena I observe fit a whole host of other known patterns. Sometimes they do, sometimes they don’t, sometimes they are just good approximations that make the pattern useful. We can analyze this patterned and numbered world and think of it as an exploration of a sort, but I would argue we are just in imaginary space until we actually have empirical evidence to support that pattern. The imaginary space is extremely useful, but I wouldn’t say it’s real, or exists. Not sure if I have enough of the gift of language to explain what I mean clearly! lol

    Liked by 1 person

    1. Thanks Swarn!

      I’m fine with Harris being brought up, but I don’t find him a particularly credible source. On Peteron’s criteria of usefulness for truth, I think we have to be careful here. A concept can be useful in a lot of ways and not be true. Some beliefs might be emotionally useful but not accurate in terms of reality. To me, the usefulness has to be epistemic usefulness, that is, something that helps us understand things. The more useful in that way something is, the greater claim I think it has on being true, or at least existing in some manner.

      Douglas Adams was a brilliant guy. I don’t think I’ve ever read that talk. I’ll have to take a look at it at some point.

      “What it really means though is that we haven’t really figured out what’s true yet, so let’s go with what is useful for now. ”

      I actually think it’s worse than that. We only ever figure out what’s true by what’s epistemically useful or not useful, in other words, by what enhances the accuracy of our predictions and what doesn’t. If there’s another measure of truth, I don’t know what it is. Note that this applies to how to reach your refrigerator and where the milk is in it as much as predicting weather or understanding the atom. It’s just that your models of how to get to the refrigerator are much more primal than ones you might know about tornadoes.

      I think I get your point about pattern manipulation and models, and how they’re not real until they’re tested for their accuracy, so your language was fine!

      Where I live, hurricanes are frequently a thing, and there are lots of models that attempt to predict where they might go. Everyone has just reached the point where they look for where most of the models are clustering and ignore the outliers. In many ways, meteorology strikes me as a science that has come to terms with the limitations of what it can predict. It does so in terms of probabilities. I’ve often wondered if the social sciences, particularly economics, with all the complexity they deal with, couldn’t learn some things from meteorology.

      Liked by 1 person

      1. Yeah, I can’t think of any field that uses modeling quite as extensively as atmospheric science and so I do think that you’re right in that the community is much more realistic about limitations. Those limitations are imposed by the physics themselves, but also just the density of measurements spatially and temporally. The trade off of course is economic in nature. Is the increased accuracy worth the increased cost of having a higher density of initial data to run the model.

        I actually think it’s worse than that. We only ever figure out what’s true by what’s epistemically useful or not useful, in other words, by what enhances the accuracy of our predictions and what doesn’t. If there’s another measure of truth, I don’t know what it is. Note that this applies to how to reach your refrigerator and where the milk is in it as much as predicting weather or understanding the atom. It’s just that your models of how to get to the refrigerator are much more primal than ones you might know about tornadoes.

        I didn’t mean to imply that there is something more epistemically useful than the scientific method in terms of enhancing the accuracy of our predictions. The example that Douglas Adams uses in his speech is this example of an island that grows crops according to a temple calendar, and when science got involved they tried to tell the inhabitants that they could increase yields, and for a short time they did, until something went out of whack in the ecosystem, and eventually the inhabitants went back to their temple calendar and things returned to normal. So this is what I meant about some things being useful, because science is missing something here, that the temple calendar isn’t. The problem being that the temple calendar doesn’t know why it has it right. The temple calendar, while centered around their religious mythology is still likely developed over centuries of observation and so science is yet to derive a trend or a detail that the temple calendar has. So for at least the moment something that is useful (the temple calendar) wins until science can figure out how to not only improve yields, but also improve yields long term. So I guess I can simply imagine other examples like this where perhaps science hasn’t asked the right questions, and so less reliable methods of “knowing” might in the short term be more useful than the scientific method. But clearly in the long-term science would eventually win out.

        Like

        1. “I didn’t mean to imply that there is something more epistemically useful than the scientific method in terms of enhancing the accuracy of our predictions.”

          Sorry, I didn’t mean to imply that you did.

          “The problem being that the temple calendar doesn’t know why it has it right.”

          I do think we should challenge age old wisdom with care. It’s always possible that, like it in this situation, it’s right because it evolved to be right. It can be operationally right (giving the right advice on when to plant) without having the right model as to its rightness (because the gods, ancestors, or whatever say so).

          To use Dennett’s phrase, there can be competence without comprehension. We see it all the time in the animal kingdom. It’s very unlikely that worker ants know why they’re doing what they do, or squirrels comprehend why they save nuts. Their action patterns are too fixed for it to be anything they individually develop.

          Even in science, we have to be careful. Sometimes the narrative on why a theory works is dead wrong. Ptolemy had a mathematical model of the universe that accurately predicted naked eye observations, but the explanation for why it worked was utterly wrong. Put another way, the narrative made predictions that couldn’t be tested until telescopes came along.

          This is one of the reasons I lean toward instrumentalism. Aspects of a theory that haven’t been tested can always turn out to be wrong.

          Liked by 2 people

    2. “I actually think it’s worse than that. We only ever figure out what’s true by what’s epistemically useful or not useful, in other words, by what enhances the accuracy of our predictions and what doesn’t. If there’s another measure of truth, I don’t know what it is.”

      I agree counselor, it is actually much worse than you can even imagine. Your musing: “If there’s another measure of truth, I don’t know what it is.” reminds me of a conversation I recently had with an individual who was fixated with rationality. Like yourself, the individual kept referring back to the default reference point: “If not rationality, then what?” The two are very similar, yes? If not the same thing.

      The other measure of truth is simple Mike: “Know Thyself”. It is a motto inscribed on the frontispiece of the Temple of Delphi. If one does not know thyself, one does not know any thing at all. The accuracy of predictions contributes nothing to our understanding of the true nature of reality, neither does rationality. When the absurdity of predictions and rationality has run their full course, all of which ultimately culminate in absurdity of one form or another, the only thing left to offer upon the sacred alter of rationality are coy anecdotes. Playing in the sandbox, that’s all we are doing.

      Get out of the sandbox and do the really hard work: “Know Thyself” and you will have within your grasp unprecedented power which will lead to unprecedented understanding….

      Like

      1. Lee,
        So, how else can we know our self except by having predictive models of our self? What other kind of knowledge is there? Can you give me any examples of it?

        How should one go about doing the hard work? What are the actionable steps?

        Like

        1. Here’s the postulate Mike: If neither materialism nor idealism* are true, one is left with a Reality that is for all intensive purposes, eerily quiet yes? Now, if that is where the rubber meets the road, then according to Parmenides, one must be willing to listen to the “still” heart of persuasive reality, and then be persuaded by that reality. Choosing that path is the hard work. It’s a tough decision, a decision that most people are unwilling to make. That is the first step in a progression of getting to “Know Thyself”.

          Now, if one chooses to cling to the construct of materialism or idealism, then all bets are off. As a consequence, the solipsistic self-model remains the center of the “known” universe. I’m sorry Mike, but that visual is one screwed up paradigm…

          ______
          *All religions, both eastern and western in flavor fall under the classification of idealism.

          Like

          1. “then according to Parmenides, one must be willing to listen to the “still” heart of persuasive reality”

            But what does that mean? When I sit quietly and contemplate reality, I find the view I laid out in the post to be the one that makes sense. So where am I going wrong?

            Like

          2. “Know Thyself”: As a solipsistic self-model, you are your own reference point. You, and you alone decide what every thing means, you and you alone hold the reins of power. The only question remaining is: What will you do with that power. Please refer to the twelve step program for addicts. Because believe it or not, agree or disagree, like it or not, human beings without exception are all hopeless, helpless addicts. Addiction is the core human nature. We are addicted to rationality because of the control it imparts, and the substance of our abuse is “power”.

            (My brother-in-law once told me that he was intrigued by my insights and that he could accept my assessment of a core human nature if it was an analogy. So far so good right, that was until I told him it wasn’t an analogy.)

            Like

          3. I don’t know Lee. Saying we’re all addicted to rationality seems like saying we’re all addicted to thinking. I actually think, if anything, there should be more of that. Of course, as Hume noted, reason is the slave of the passions, it exists to serve our instinctive impulses. Its power seems to be in clearing up traffic jams between those impulses. Denying it seems like a recipe for short term hedonism.

            But maybe you mean something different than what is normally meant by “rationality”?

            Like

          4. You are relatively close in your analysis Mike. Rationality is the only tool in our toolbox, and that tool is fine tuned to serve us in our primary experience. We are totally and completely dependent upon rationality in order to navigate our environment successfully, just like we are dependent upon water, food, and oxygen for our own existence. Unlike other dependencies though, we are addicted to rationality because of the control that it imparts. And that control is integral to the self-model. Control is a painfully elusive yet powerfully dynamic abstraction. This is important and should not be dismissed: Without a sense of control there is no sense of self; so those two dynamics are coextensive, they work together and cannot be divided. It is within the architecture of self, power and control that the problem arises. Self, power and control are dynamics which are complicated in their execution, yet fundamentally simple in principle.

            We all agree that there is something seriously wrong with human beings, yet no one is able to isolate that defect with any specificity. Well, I just did. There is no way to expand on the subject in a forum like this, but I dedicate several chapters to addiction in my book. When considering the hypothesis, one has to have a context, so I like to refer to what Dr Phil McGraw says about addiction: Addiction is not a problem unless it interferes in one’s ability to live a normal, productive life. Or as Robert Pirsig would say: rationality is not problematic unless it interferes with the nature progression of static patterns as they discover and move toward dynamic quality. Which simply means understanding our world and ourselves.

            Like

  13. Interesting to see well-trodden ground trampled. To paraphrase philosophers through time:
    – 0 is one, one is two, you know the drill.
    – To identify is to contextualize is to have transcended already.
    – Existence precedes essence.
    Maybe you are right; metaphysics makes no progress.
    But neither does anything.

    Like

  14. We often tend to forget about Telic in the classical sense when considering Epistemological and Onotological questions, particularly those that collide or contradict, challenge or otherwise compliment Agency in an historical sense, as Experience per se. I like to think about the Conflict as one between Objectivity and Subjectivity, which I take to be the two dominant and fundamental principles of Perspective in the Existential Field. Agency is the Key, and Instinct (subjectivity)/Reason (objectivity) throw the dice in a game of Roulette. Die free today by Instinct, or serve tomorrow under Reason seems to be the Order of Operations, outside the heroes of Heart.

    Liked by 1 person

    1. I actually don’t view subjectivity and objectivity as being in conflict. For me, it’s worth remembering that all we ever have direct access to is our own subjective experience. The objective we can only infer through theories, reasoning. But we can asses the reliability of those theories by how predictive they are for future subjective experiences.

      Sometimes those theories are so predictive that they cause us to doubt some of our direct subjective intuitions. This is totally valid. It’s why we evolved the ability to reason. Reason’s adaptive “purpose”, its telic if you will, seems mostly to veto instinctive impulses that foresight tells us we shouldn’t engage in.

      Liked by 1 person

      1. That’s often true, of your last words. While instinct is not just ‘impulse’ nor the irrational, blind affectation of the will, or of Agency; but its very center and circumference. Reason enters when things need get done, and an accountant takes the reigns to see that it does…get done. The ‘motivation’ inbetween the potential and actual Agency, is the very subjective grounds of conscience itself, by and through which we see, understand and comprehend all elements of our experience and the Existential Field, past-present-future withal. I’ll revisit your earlier assertion in due time, which I think bears some clarification, if not objection. Thanks for engaging.

        Liked by 1 person

  15. Nope, not a thing.
    I think Jared Diamond basically got it right in, “The Worst Mistake…”, but I am not totally pessimistic.
    We can complete our understanding of the issues, but the issues remain. We will not transcend them.

    Like

    1. Diamond in his book, ‘The World Until Yesterday’ also indicated that native people, including hunter gatherers, who are given access to modern lifestyles, generally adopt them pretty rapidly. They don’t seem to be sentimental about traditional lifestyles. That seems to indicate something progressive has happened with civilization.

      But humans aren’t wired to be happy except for brief periods. Instinctive emotions evolved to spur action, and ongoing satisfaction wouldn’t be adaptive. We’d have to rewire the brain to overcome that.

      Like

      1. I think that in most cases, they are given an offer they can’t refuse, and are as pragmatic as any of us.
        From where I stand, it has not worked out too well, and there seems to be widespread recognition of that fact among the descendants of aboriginal people.
        I agree with you, but is happiness really what we are about? Unless you are one of those smug utilitarians.

        Like

  16. I’m reading The Order of Time, by Carlo Rovelli, and he just did a page on what is “real” or can be said to “exist”…

    “There are so many different usages of the verb, different ways in which we can say that a thing exists: a law, a stone, a nation, a war, a character in a play, the god (or gods) of a religion to which we do not belong, the God of the religion to which we do belong, a great love, a number. . . . Each one of these entities “exists” and “is real” in a sense different from all the others. We can ask ourselves in what sense something exists or not (Pinocchio exists as a literary character but not as far as any Italian register office is concerned), or if a thing exists in a determined way (does a rule exist preventing you from “castling” in chess, if you have already moved the castle?). To ask oneself in general “what exists” or “what is real” means only to ask how you would like to use a verb and an adjective. It’s a grammatical question, not a question about nature.”

    Ultimately, the word is as freighted and fraught as the word “consciousness.”

    As for small “p” platonism, I think math shows it’s true to the extent its being true really means anything. The lawfulness of reality does extend to the structure of thought.

    Like

      1. 😀 In Through Two Doors at Once, which is all about the two-slit quantum thing, author Ananthaswamy wraps up by comparing our unresolved understanding to the two-slit experiment itself: the matter is like the particle in flight, apparently going through two doors at once. At this point we just don’t know which is right. Could be either.

        Great metaphor, I thought.

        Liked by 1 person

  17. I made it through a quarter of that SEP article on this “new” lower case sort of platonism, though that was about all I could stand. Tell me Mike, are my own sentiments expressed by any of those distinguished academics? Quine and such? Or am I perhaps missing something? To me this just doesn’t seem very hard.

    As I see it “redness”, “seven”, and indeed all humanly fabricated terms, can only exist as a product of the conscious entities which happen to possess such faculties. Take away subjects for which a given term means anything, and the term should no longer exist — not in spacetime, and not anywhere else. Otherwise there are only causal dynamics, or even supernatural dynamics, but no “redness”, no “seven” and no other constructs of language. I’m saying that humanly fabricated terms are merely a product of humans. Thus it seems to me that all forms of platonism, exists as standard waste of time philosophy. Did anyone mention anything like that in the SEP article?

    Like

    1. I can’t say whether Quine espouses any of your positions. I’m not that familiar with his overall work. I would note that the paper I shared on the next post discusses the possibility of a virtual machine structure existing on top long standing evolved framework, and cites Ned Block for that view, which sounds like it might have some similarities to your second computer.

      Sounds like you’re in the nominalist camp. At the moment, so am I. Although if you think the answer is obvious, I suspect you haven’t thought it through. That said, this is definitely metaphysics, which means its practical effects, if any, aren’t obvious. SEP doesn’t dismiss those types of topics. It’s worth noting that what is metaphysics today might be science in the future. (See the history of atoms.)

      Liked by 1 person

      1. I really shouldn’t dismiss traditional philosophy as a waste of time. In the western world humanity has been having lots of academic fun with it for two and a half millennia. Thus philosophy shouldn’t be any more of a waste of time that our various arts have been (and no, I’m not a true philistine!). But I also get frustrated with the modern field because I believe that it both can and must help the institution of science improve. Without effective agreed upon principles of metaphysics, epistemology, and axiology, or the very domain of philosophy, it seems to me that science suffers all sorts of maladies, and most extensively in its mental and behavioral disciplines.

        So what’s my solution? I’d have us keep traditional philosophy just as it is. But I’d also like a new society of philosopher to emerge that has no use for tradition, culture, or academic entertainment in general. The only purpose of this society would be to develop apparently effective agreed upon principles from which to do science better. If scientists were to find its principles helpful, then this new society should grow to become an integral part of science itself.

        As far as this modern platonism business goes, my simple solution shall remain until valid criticism of it can be produced. This is to say that all words exist as a causal product of something which is able to consciously grasp them — not otherwise. (I highly doubt that my proposed new variety of philosopher would find human centric notions such as platonism, all that notable.)

        Mike, thanks for looking out for potential associations with my dual computers model of brain function. That paper isn’t really my cup of tea, but is there a specific part that you can direct me to?

        Liked by 1 person

        1. Eric,
          On productive epistemology, my own view is summarized in the second paragraph of the post.

          My actual commitment is empiricism. By “empiricism” here, I don’t necessarily mean physical measurement, but conscious experience, specifically reproducible or verifiable experience, and inferred theories that can predict future experiences, with an accuracy better than alternate theories, or at least better than random chance.

          I perceive that this is actually fairly common in science oriented people, although perhaps not always in the all encompassing manner I hold it.

          On the paper, it’s really only briefly mentioned, but it does cite this paper for the view:
          Block, N. (1995). The mind as the software of the brain. In E. E. Smith & D. N. Osherson (Eds.), Thinking: An invitation to cognitive science (pp. 377–425). Cambridge: MIT Press.

          This may be an online version: https://www.nyu.edu/gsas/dept/philo/faculty/block/papers/msb.html

          Like

  18. Mike said: “It seems to me that a much better approach would be to argue that they’re emergent from something. Of course, that requires identifying what they emerge from.”

    Bingo! To solve the riddle of emergence, all one has to do now is substitute Reality for the Kingdom of God in the following quote, and then search for that “seed” from which every “thing” emerges. But if one finds Kant’s reality/appearance distinction, an ontology which distinguishes the noumena (Reality) from the phenomena (Appearance of Reality) offensive, then all bets are off. One will have to settle for the paradox of dualism or illusionism and be content to just suck on it.

    “How will we liken the Kingdom of God? Or with what parable will we illustrate it? It’s like a grain of mustard seed, which, when it is sown in the earth, though it is less than all the seeds that are on the earth, yet when it is sown, grows up, and becomes greater than all the herbs, and puts out great branches, so that the birds of the sky can lodge under its shadow.”
    — Mark 4:30–32, World English Bible*

    _____
    *Wikipedia

    Like

Your thoughts?

This site uses Akismet to reduce spam. Learn how your comment data is processed.